Preview (15 of 89 pages)

Chapter 10 Producing Goods and Services MULTIPLE CHOICE QUESTIONS. Choose the one alternative that best completes the statement or answers the question. 1) Which of the following is correct with regard to the opening case entitled “Too Many Recalls”? A) The recalls of so many Toyota vehicles forced the car maker into bankruptcy B) Millions of toys that were made in China for Mattel have had to be recalled, and the main blame was found to be poor workmanship by the Chinese C) As a result of Toyota's car recalls, it lost its top spot in terms of customer loyalty D) An analysis of the Mattel toys that were recently recalled revealed that the problem was in Chinese manufacturing methods, not Mattel's design E) All of these are correct Answer: C 2) The term service operations refers to A) production activities that yield tangible products. B) production activities that yield tangible and intangible service products. C) firms that make tangible products. D) service activities that yield intangible products. E) service activities that yield tangible products. Answer: B 3) Production activities that yield both tangible and intangible service products are known as A) the Industrial Revolution. B) service operations. C) goods production. D) the factory system. E) scientific management. Answer: B 4) Which of the following is correct with regard to the boxed insert entitled "Open Source Automobile Manufacturing"? A) The open source concept does not work for the manufacturing of automobiles because the automobile is such a complex product B) The open source concept works well for the mass production of automobiles C) Open source automobile manufacturing does not go beyond the design phase D) Local Motors has already demonstrated the feasibility of open source automobile manufacturing because it has sold over 500,000 cars E) Open source manufacturing involves first posting a challenge for a certain car component, and then inviting engineers and designers to upload sketches and ideas for the component Answer: E 5) The clerk at the 7-Eleven store and the morning radio announcer are examples of people who work in ___________, while companies that make things like radios and automobiles are engaged in __________. A) minimum wage operations; maximum wage operations B) goods production; service operations C) service operations; goods production D) high-contact operations; low-contact operations E) none of the above Answer: C 6) Which of the following is correct with regard to the boxed insert entitled "The Unicycle Motorbike"? A) The inventor decided his new product had to be "cool" in addition to providing transportation B) The product is environmentally friendly, but that was not a major reason motivating its development C) The product can travel at speeds up to 120 kilometres per hour D) By carefully managing resources, the inventor has been able to develop the product by spending just over $10,000 E) All of these are correct Answer: A 7) Suppose that you have no clean clothes, but you have a job interview tomorrow. You normally never use any other type of laundry detergent other than your regular brand. You find that it is currently sold out at your local convenience store, so you decide to buy the store brand because it best satisfies your need for A) possession utility. B) place utility. C) time utility. D) interval utility. E) ownership utility. Answer: C 8) Time utility is created by making a product A) with a form that consumers desire. B) which will retain a high resale value over a period of time. C) available in a place convenient for customers. D) that customers can take pleasure in owning E) available at a moment when consumers want it. Answer: E 9) When a company turns out Christmas ornaments so that they are available in the stores in November, it has created A) time utility. B) price utility. C) ownership utility. D) place utility. E) form utility. Answer: A 10) By shipping goods six to eight weeks after they are ordered, a firm is hindering A) time utility. B) price utility. C) ownership utility. D) place utility. E) form utility. Answer: A 11) A product's ability to satisfy a human want is called A) utility. B) usefulness. C) customer satisfaction. D) marketability. E) demand. Answer: A 12) Which of the following is not a form of utility? A) Form utility B) Price utility C) Place utility D) Time utility E) Ownership utility Answer: B 13) If Toro introduces a new snow blower in April, it has ignored A) price utility. B) ownership utility. C) place utility. D) time utility. E) form utility. Answer: D 14) By turning raw materials into finished goods, production creates A) ownership utility. B) price utility. C) form utility. D) place utility. E) time utility. Answer: C 15) When your favourite restaurant prepares your favourite meal, they are creating A) form utility. B) price utility. C) time utility. D) place utility. E) ownership utility. Answer: A 16) Making products available to consumers when they want them is creating A) ownership utility. B) time utility. C) place utility. D) form utility. E) price utility. Answer: B 17) The quality of a product that satisfies a human need during its consumption is known as A) ownership utility. B) place utility. C) need utility. D) form utility. E) time utility. Answer: A 18) In many parts of the world, public washrooms are available on a pay per use basis. What form of utility does this service maximize most to its users? A) Place utility B) Form utility C) Interval utility D) Time utility E) Ownership utility Answer: A 19) Select the best explanation of the term "place utility." Place utility is created by making a product A) available at a location that is convenient for customers. B) that customers can take pleasure in owning. C) different from all other products. D) available at a time when consumers want it. E) which will retain a high resale value over a period of time. Answer: A 20) Domino's original strategy of delivering pizzas in "30 minutes or it's free" created a competitive advantage based on A) price utility. B) time utility. C) place utility. D) form utility. E) ownership utility. Answer: B 21) When steel is turned into a finished good like a car, ________ utility has been created. A) place B) time C) interval D) ownership E) form Answer: E 22) A soft drink manufacturer who has many outlets for its product is providing A) place utility. B) ownership utility. C) form utility. D) price utility. E) time utility. Answer: A 23) The type of utility provided by convenience stores such as 7 Eleven and Circle K is A) price utility. B) form utility. C) product utility. D) ownership utility. E) place utility. Answer: E 24) By making a product available for consumers to use, ________ is created. A) price utility B) place utility C) ownership utility D) time utility E) form utility Answer: C 25) By accepting major credit cards a retail outlet has created A) price utility. B) time utility. C) ownership utility. D) form utility. E) place utility. Answer: C 26) By accepting only cash for purchasing merchandise, a retail outlet has hindered A) ownership utility B) place utility. C) price utility. D) time utility. E) form utility. Answer: A 27) By selling their products to only a few selected retailers, the producer is hindering A) ownership utility. B) form utility. C) place utility. D) price utility. E) time utility. Answer: C 28) When a company uses an intensive distribution strategy, it is facilitating _________ utility. A) Ownership utility B) Form utility C) Place utility D) Interval utility E) Time utility Answer: C 29) The type of utility provided by convenience stores such as Mike's Milk and Country Style is A) time utility. B) price utility. C) ownership utility. D) place utility. E) form utility. Answer: D 30) When raw materials are converted into finished goods, ________ utility has been created. A) form B) time C) ownership D) place E) none of these Answer: A 31) In the process of assembling a new car as it moves down the assembly line, General Motors has created A) price utility. B) ownership utility. C) form utility. D) place utility. E) time utility. Answer: C 32) By making a product that customers can take pride in having, production creates A) resale value utility. B) form utility. C) time utility. D) operational utility. E) ownership utility. Answer: E 33) Which term is used to identify the systematic direction and control of the processes that transform resources into finished goods and services? A) Quality assurance management B) Marketing management C) Operations management D) Financial management E) Purchasing management Answer: C 34) Steve manages people who take various raw materials and pre-made assemblies and transforms them into a unique product that is sold to retail stores. Steve is involved in A) service creation. B) goods creation. C) manufacturing management. D) plant management. E) operations management. Answer: E 35) One of the key differences between service and manufacturing operations is A) manufacturing products are characterized by intangibility, and customization. B) manufacturing operations feature a unique link between process and outcome. C) manufacturing operations are less storable. D) services are performed, but goods are produced. E) services focus on the outcome of the production process. Answer: D 36) Which of the following are unique aspects that separate services from goods? A) Intangibility and warranties B) High-contact and low-contact C) Time and place utility D) Intangibility and unstorability E) Form and time utility Answer: D 37) If an Air Canada flight from Toronto to Calgary has 20 empty seats, this illustrates the unique service characteristic of A) intangibility. B) unstorability. C) demand. D) customization. E) oversupply. Answer: B 38) Service products are characterized by three key qualities A) intangibility, uncustomized, unstorability. B) intangibility, customization, unstorability. C) intangibility, customization, storability. D) tangibility, customization, storability. E) tangibility, uncustomized, storability. Answer: B 39) Which term identifies services that cannot be described by their physical attributes? A) Invisible B) Perishable C) Intangible D) Produced E) Customized Answer: C 40) You have gone to the doctor to check out some recent pains. The quality of her expertise to find and diagnose the problem is an illustration of the unique service characteristic of A) intangibility. B) warrantees. C) customization. D) perishability. E) past experience. Answer: A 41) Services are ________ and cannot be touched, tasted, smelled, or seen. A) unstorable B) flexible C) customized D) intangible E) leveraged Answer: D 42) Services are ________, and are designed to meet the specific needs of the purchaser. A) unstorable B) leveraged C) intangible D) customized E) flexible Answer: D 43) When you have your hair cut and styled the way you like it, it illustrates the unique service characteristic of A) unstorability. B) customization. C) past experience. D) intangibility. E) warranties. Answer: B 44) Of the following products/services, which is the most intangible? A) Restaurant meal B) Haircut C) Jewellery D) Legal services E) Clothing Answer: D 45) Of the following products/services, which is the most tangible? A) Child care B) A restaurant meal C) Car repair D) Clothing E) Dental work Answer: D 46) A(n) ________ process breaks down resources into components. A) synthetic B) assembly C) fabrication D) analytic E) chemical Answer: D 47) Tyson reduces whole chickens into packaged parts through a(n) ________ process. A) chemical B) fabrication C) synthetic D) analytic E) assembly Answer: D 48) The set of methods and technologies used in the production of a good or service is a(n) ________ process. A) manufacturing B) form creation C) product D) management E) operations Answer: E 49) Which of the following is a method of classifying an operations process? A) By the purchasing method used to obtain resources B) By the service processes C) Survey of owners to determine feedback D) By the fabrication processes E) By the transformation process that is used Answer: E 50) This transformation process alters the basic composition of raw materials and is used in industries like aluminum, steel, fertilizer, petroleum, and paint. It is the A) chemical process. B) fabrication process. C) transport process. D) clerical process. E) assembly process. Answer: A 51) What types of processes are used in the manufacturing of fertilizer? A) Assembly processes B) Transport processes C) Fabrication processes D) Chemical processes E) Clerical processes Answer: D 52) Which of the following transformation techniques mechanically alters the basic shape or form of the product? A) Chemical process B) Quality process C) Fabrication process D) Assembly process E) Transport process Answer: C 53) This transformation process mechanically alters the basic shape or form of a product and is common in industries like metal forming, woodworking, and textiles. It is the A) clerical process. B) assembly process. C) chemical process. D) transport process. E) fabrication process. Answer: E 54) What kind of transformation occurs in an assembly process? A) Various components are transported from one location to another B) Data is combined into useable information for a management report C) Various components are put together to create a new product D) Finished products are taken apart for inspection E) Raw materials are mechanically altered Answer: C 55) This transformation process involves goods being sent from manufacturing sites to consumers through warehouses to retail locations. It is the A) fabrication process. B) chemical process. C) clerical process. D) assembly process. E) transport process. Answer: E 56) This transformation process combines data on such items as employee absences, machine breakdowns, and inventory reports. It is the A) assembly process. B) fabrication process. C) chemical process. D) transport process. E) clerical process. Answer: E 57) An artist working with steel uses which type of process the most? A) Transport processes B) Clerical processes C) Fabrication processes D) Assembly processes E) Chemical processes Answer: C 58) Someone who is able to creatively and efficiently turn raw data into useable managerial reports uses A) analytic processes. B) analytical processes. C) clerical processes. D) transport processes. E) assembly processes. Answer: C 59) This transformation process puts together various components and is common in the electronics, appliance and automotive industries. It is the A) clerical process. B) transport process. C) chemical process. D) fabrication process. E) assembly process. Answer: E 60) The production process which combines a number of raw materials into a new finished product is known as A) creating place utility. B) analytic process. C) transformation process. D) synthetic process. E) creating time utility. Answer: D 61) The conversion of meat on the hoof to a T-bone steak is a(n) A) continuous process. B) synthetic process. C) administrative process. D) analytic process. E) chemical process. Answer: D 62) How does an analytic production process work? A) It breaks down the basic resources into components B) A cost analysis is made to determine the best way to ship the product to another location C) It combines a number of raw materials into a finished product D) It puts together several components E) Raw materials are chemically altered Answer: A 63) Which production process breaks down basic resources into components? A) Synthetic process B) Analytic process C) Chemical process D) Analytical process E) Assembly process Answer: B 64) Lumber companies convert raw lumber into dimensional lumber and paper products through a(n) ________ process. A) synthetic B) fabrication C) assembly D) analytic E) chemical Answer: D 65) In a(n) ________ process, resources are combined to create finished products. A) fabrication B) analytic C) chemical D) synthetic E) assembly Answer: D 66) GE produces refrigerators through a(n) ________ process. A) chemical B) synthetic C) assembly D) analytic E) fabrication Answer: C 67) The type of service process in which the customer must be an active participant is a(n) ________ system. A) high-contact B) active C) low-contact D) high-interaction E) low-interaction Answer: A 68) The type of service process in which the customer need not be an active part of the system to receive the service is a ________ system. A) high-contact B) high-interaction C) low-interaction D) low-contact E) passive Answer: D 69) Suzy is going to her hairdresser to get her hair cut and set. This type of service system is a(n) ________ system. A) low-interaction B) high-contact C) low-contact D) active E) high-interaction Answer: B 70) Public transportation systems are ________ systems. A) high-interaction B) low-interaction C) low-contact D) active E) high-contact Answer: E 71) Carolyn's car needs a tune-up so she has taken it to her favorite mechanic for repair. This type of service system is a ________ system. A) low-interaction B) passive C) high-interaction D) low-contact E) high-contact Answer: D 72) Cheque processing at your local bank is an example of a(n) ________ system. A) active B) high-contact C) high-interaction D) low-interaction E) low-contact Answer: E 73) A public transit system is an example of a _________ process, while a cheque processing operation at a bank is a __________ process. A) fabrication; chemical B) transport; fabrication C) high contact; low contact D) low contact; high contact E) goods producing; service producing Answer: C 74) In terms of business strategy as a driver of operations, Toyota uses the _________ strategy, while 3M uses the _________ strategy. A) quality; dependability B) flexibility; dependability C) quality; flexibility D) lower price; quality E) lower price; flexibility Answer: C 75) In terms of business strategy as a driver of operations, 3M uses the _________ strategy, while FedEx uses the _________ strategy. A) quality; dependability B) flexibility; dependability C) quality; flexibility D) lower price; quality E) lower price; flexibility Answer: B 76) In terms of business strategy as a driver of operations, Save-A-Lot uses the _________ strategy, while Toyota uses the _________ strategy. A) quality; dependability B) flexibility; dependability C) quality; flexibility D) low price; quality E) low price; flexibility Answer: D 77) Because FedEx emphasizes __________, it operations capabilities emphasize ______________. A) quality; quality control procedures B) dependability; wireless information system to track packages C) low prices; cost control D) flexibility; excess production capacity E) dependability; quality control procedures Answer: B 78) You are creating an operations planning and control system for a manufacturer. What should your first step be? A) Conduct production control B) Develop a long-range production plan C) Send output to customers D) Develop production schedules E) Review the company's business plan Answer: E 79) What is the long-range production plan based upon? A) The quality assurance plan B) The number of production plants now available C) The overall business plan D) The hourly wage rate of factory workers E) The proximity to raw materials and markets Answer: C 80) Which of the following items would not be included in the long range production plan? A) The number of plants needed to meet the forecasted demand B) The technology, labour, and machinery needed to meet forecasted demand C) The transportation and storage facilities that will be needed D) How resources will be obtained E) The monthly production schedule to meet shipping requirements Answer: E 81) What is the most logical sequence of steps in a production planning and control system? A) Business plan, long range operations plan, operations control, operations schedules, output to customers B) Operations control, operations schedules, output to customers, long range operations plan, business plan C) Business plan, operations control, operations schedules, long range operations plan, output to customers D) Business plan, long range operations plan, operations schedules, operations control, output to customers E) Business plan, operations control, operations schedules, output to customers, long range operations plan Answer: D 82) What is one reason why it is a bad idea for a company to have excess capacity? A) Wastes raw materials B) Increases customer dissatisfaction C) Ties up money in extra machines D) Decreases form utility E) Slows delivery of product Answer: C 83) What is an option for companies that want to have a capacity to meet peak demand? A) Plan for peak demand by having, for example, all cash registers open 3 days before Christmas B) Use a qualitative forecast to determine average demand C) Attempt to meet most, but not all, peak demands D) Plan for normal demand E) Lower prices of the products that are being produced Answer: A 84) Capacity planning means ensuring that a firm’s capacity ________ the normal demand for a product. A) is substantially less B) is equal to C) is substantially more D) is slightly less E) is slightly more Answer: E 85) In low-contact systems, capacity should be set ____________ the average level of demand, while in high-contact systems managers must plan to meet __________ demand. A) less than; peak B) more than; peak C) equal to; average D) equal to; peak E) less than; average Answer: D 86) In businesses like restaurants or supermarkets the business is sized to meet a level of A) normal demand. B) low demand. C) peak demand. D) average demand. E) high demand. Answer: C 87) Mega Computers services and upgrades the computers that it sells. In sizing its service operations it should use A) peak demand. B) average demand. C) low demand. D) high demand. E) normal demand. Answer: B 88) The number of customers that Bob's Barbecue serves varies considerably from hour to hour. In determining the number of cash registers to install they should use A) average demand. B) peak demand. C) low demand. D) high demand. E) normal demand. Answer: B 89) The amount of a product that a company can produce under normal working conditions is its A) capability. B) scheduled capacity. C) scheduled capability. D) capacity. E) normal capacity. Answer: D 90) Most firms plan to be able to produce ________ product than is normally demanded. A) substantially more B) slightly less C) slightly more D) the same amount of E) substantially less Answer: C 91) Which type of decision is most influenced by the development of industrial parks? A) Layout planning B) Capacity planning C) Location planning D) Quality planning E) Product marketing Answer: C 92) The most important factor in location planning for high-contact businesses is ___________, but for low-contact business the most important factor in location planning is ____________. A) availability of labor; proximity to customers B) proximity to customers; community living conditions C) community living conditions; transportation costs D) transportation costs; proximity to resource supplies E) proximity to customers; proximity to resource supplies Answer: E 93) In a(n) ________ layout, equipment and people are grouped together according to their function. A) assembly line B) process C) product D) customer-oriented E) flexible Answer: B 94) The layout of a production facility with equipment and people grouped together according to their function is known as A) process layout. B) customer-oriented layout. C) flexible functional layout. D) assembly line layout. E) product layout. Answer: A 95) Which of the following is best suited for a process layout? A) A food processing plant B) A clothing manufacturer C) A dry cleaning shop D) A car manufacturer E) An electronics manufacturer Answer: A 96) Machine, woodworking, and dry cleaning shops are most likely to use which layout? A) Process B) Fixed-position C) Customer D) Cellular E) Product Answer: A 97) A process layout is A) a production system in which a single factory uses automation to produce a wide variety of products. B) a spatial arrangement of production facilities designed to move families of products through similar flow paths. C) a method for analyzing a service by showing the flow of processes that constitutes it. D) a spatial arrangement of production activities designed to make one type of product in a fixed sequence. E) a spatial arrangement of production activities that groups equipment and people according to function. Answer: E 98) A cellular layout is A) a production system in which a single factory uses automation to produce a wide variety of products. B) a spatial arrangement of production activities that groups equipment and people according to function. C) a method for analyzing a service by showing the flow of processes that constitutes it. D) a spatial arrangement of production activities designed to make one type of product in a fixed sequence. E) a spatial arrangement of production facilities designed to move families of products through similar flow paths. Answer: E 99) A product layout is A) a spatial arrangement of production activities designed to make one type of product in a fixed sequence. B) a spatial arrangement of production activities that groups equipment and people according to function. C) a method for analyzing a service by showing the flow of processes that constitutes it. D) a spatial arrangement of production facilities designed to move families of products through similar flow paths. E) a production system in which a single factory uses automation to produce a wide variety of products. Answer: A 100) In a ________ layout, resources move through a fixed sequence of steps to become finished goods. A) process B) customer C) fixed-position D) product E) cellular Answer: D 101) The layout that is efficient and inexpensive because it simplifies work tasks and uses unskilled labor is the ________ layout. A) customer B) process C) product D) cellular E) fixed-position Answer: C 102) In a custom cake bakery, blending of ingredients is done in the mixing area, baking occurs in the oven area, and decorations are applied in the finishing area. This is which layout? A) Cellular B) Customer C) Process D) Fixed-position E) Product Answer: C 103) A clothing manufacturer that sets up an area for making different kinds of pockets, another area for sleeves, and another for collars, is using the ________ layout. A) product B) fixed-position C) process D) customer E) cellular Answer: E 104) Which of the following terms refers to the spatial arrangement of production facilities designed to move families of products through similar flow paths? A) Fixed position layout B) Flexible manufacturing system C) U-shaped production line D) Cellular layout E) Flow layout Answer: D 105) Which layout is most likely to depend upon general-purpose machinery and skilled labour to perform a variety of jobs for different customers? A) Process B) Fixed-position C) Customer D) Cellular E) Product Answer: A 106) A job shop would be most likely to use a ________ layout. A) process B) fixed-position C) product D) customer E) cellular Answer: A 107) In which of the following layouts do resources move through a fixed sequence of steps to become finished goods? A) Product B) Process C) Customer D) Cellular E) Fixed-position Answer: A 108) An assembly line would be most likely to use a ________ layout. A) cellular B) customer C) process D) product E) fixed-position Answer: D 109) The typical layout in automobile, food-processing, and computer assembly plants is the ________ layout. A) product B) cellular C) process D) customer E) fixed-position Answer: A 110) All things considered, the _________ layout is best. A) customer B) process C) cellular D) product E) none of these Answer: E 111) ________ layouts are typically used in ________ production. A) Product; synthetic B) Flexible; custom C) Process; mass D) Product; continuous process E) Synthetic; custom Answer: D 112) All of the following are characteristics of soft manufacturing except A) emphasizes computer software and computer networks. B) automation is used only for highly repetitive tasks. C) robots play a supporting role in the manufacturing process. D) allows for customizability of the end product. E) focuses on production machines. Answer: E 113) All of the following are main elements of operations planning except A) quality. B) methods planning. C) location. D) capacity. E) process. Answer: E 114) In which of the following layouts are people and equipment set up to produce only one type of good and are rearranged according to its production requirements? A) Product B) Cellular C) Customer D) Process E) Fixed-position Answer: A 115) A ________ layout would be most likely to use assembly lines. A) customer B) fixed-position C) cellular D) process E) product Answer: E 116) The layout where the arrangement of production facilities is designed to move families of products through similar flow paths is the ________ layout. A) cellular B) process C) customer D) fixed-position E) product Answer: A 117) A production system that allows a single factory to produce a wide variety of products is called a(n) A) assembly line. B) cellular system. C) flexible manufacturing system. D) methods improvement system. E) soft manufacturing. Answer: C 118) Nissan, Toyota, and Honda now build several different models of cars using the same basic “platform.” This is an example of A) process layout. B) soft manufacturing. C) flexible manufacturing systems. D) the movable factory. E) quality control. Answer: C 119) The step-by-step examination of procedures in order to reduce waste and inefficiency is called A) assembly line improvement. B) process improvement. C) methods improvement. D) quality improvement. E) procedures improvement. Answer: C 120) Once plans identify needed resources and how they will be used to reach a firm's goals, managers must develop timetables for acquiring resources for production. This aspect of operations is called A) resourcing. B) planning. C) timing. D) controlling. E) scheduling. Answer: E 121) The schedule that shows which products will be produced, when production will occur, and what resources will be used during specified time frames is called a A) inventory control schedule. B) master production schedule. C) control chart. D) short-term detailed production schedule. E) material requirements plan. Answer: B 122) Which of the following would not be shown on the master production schedule? A) Identification of resources to be used B) Dates of production C) Shipping dates D) Identification of products to be produced E) How many of each product will be produced Answer: C 123) What is the meaning of "operations control"? A) Planning and organizing the flow of materials B) Purchasing the right raw materials, in the right quantity, at the right price, and at the right time C) Checking to ensure that production decisions are being implemented D) Stocking items needed for production that are themselves produced within the firm E) Monitoring production performance by comparing results with plans and schedules Answer: E 124) Which of the following is correct with regard to the box entitled “For The Greener Good”? A) Wal-Mart’s decision to conduct its business in a more environmentally-friendly way means that its suppliers have to bear the burden of meeting Wal-Mart’s new demands B) Critics of Wal-Mart concede that the company has the potential to use its power for good C) Changes made by suppliers as they attempt to meet Wal-Mart’s new environmental standards have resulted in lowered shipping costs for Wal-Mart D) Wal-Mart’s goal is to reduce packaging by 5 percent by 2013 E) All of these are correct Answer: E 125) What is the meaning of "operations control"? A) Stocking items needed for production that are themselves produced within the firm B) Purchasing the right raw materials, in the right quantity, at the right price, and at the right time C) Planning and organizing the flow of materials D) Monitoring production performance by comparing results with plans and schedules E) All of these Answer: D 126) Betty is a production supervisor in a leather goods factory. Her main responsibility is operations control. Betty would probably be most concerned with A) determining what products to produce. B) deciding how to produce products. C) deciding on product pricing. D) ensuring that production plans or schedules are met. E) deciding where to produce the products. Answer: D 127) What are the two primary sub-areas of operations control? A) Purchasing and inventory control B) Materials management and purchasing C) Just-in-time inventory systems and PERT charts D) Production process control and materials management E) Inventory control and materials inventory Answer: D 128) The process of monitoring production performance by comparing results with plans is called A) management control. B) production control. C) performance control. D) operations control. E) manufacturing control. Answer: D 129) Planning, organizing, and controlling the flow of materials from design through distribution of finished goods is A) quality control. B) material requirements planning. C) statistical process control. D) materials management. E) inventory planning and control. Answer: D 130) The use, where possible, of uniform components in the production process is A) uniform component control. B) optimized inventory control. C) component management. D) standardization. E) component optimization. Answer: D 131) Which term is used in materials management to identify the use of uniform components rather than new or different components? A) Opportunity costs B) Incrementalism C) Forward buying D) Holding costs E) Standardization Answer: E 132) Tom is concerned with the storage of both incoming materials for production and finished goods for distribution to customers. Tom's job is most likely in the ________ area. A) inventory control B) purchasing C) transportation D) warehousing E) quality control Answer: D 133) Ed's job responsibilities include the receiving, storing, handling, and counting of all raw materials. He is probably in the ________ function. A) inventory control B) transportation C) quality control D) warehousing E) purchasing Answer: A 134) Included in ________ is the receiving, storing, handling, and counting of all raw materials, partly finished goods and finished goods. A) transportation B) quality control C) inventory control D) warehousing E) purchasing Answer: C 135) John's job responsibilities include the acquisition of all the raw materials and services that the company needs to produce its products. He is most likely in the ________ function. A) warehousing B) transportation C) quality control D) inventory control E) purchasing Answer: E 136) Laine's job responsibilities include evaluating potential suppliers, negotiating terms of service and maintaining positive buyer-seller relationships. He is most likely in the ________ function. A) inventory control B) warehousing C) supplier selection D) transportation E) quality control Answer: C 137) Dean's job responsibilities include the receiving, storing, handling, and counting of all raw materials, partly finished goods, and finished goods. He is most likely in the ________ function. A) supplier selection B) quality control C) inventory control D) warehousing E) transportation Answer: C 138) What does a just-in-time production system attempt to accomplish? A) Materials and parts needed at each production step are brought together at the right time B) Attempts to control the overall production process C) Shows when production must be completed to satisfy consumer demand just-in-time D) Shows the critical path where production problems will be the most damaging to the schedule E) Breaks down the production process into steps necessary for completion Answer: A 139) Just-in-time (JIT) production systems A) may lead to increased inventory obsolescence. B) have been widely used since the Great Depression of the 1930’s. C) bring together all the needed materials and parts at the precise moment they are required for each production stage, not before. D) keep inventory levels high so deliveries to customers can be made just in time. E) increase the number of goods in process and save money by replacing stop-and-go production with smooth movement. Answer: C 140) Mount Sinai Hospital no longer keeps any inventory. It is using A) material requirements planning. B) warehousing. C) master production scheduling. D) just-in-time inventory. E) zero tolerance inventory control. Answer: D 141) The major objective of JIT is to A) minimize the number of different parts being used. B) minimize stock-outs of raw materials. C) minimize inventory costs. D) ensure smooth production operations. E) improve the quality of incoming raw materials and parts. Answer: C 142) What advantages are obtained from the use of a just-in-time inventory system? A) Suppliers can be less quality conscious and thus will have lower costs B) The production system becomes an intermittent process C) The warehousing function increases in importance D) The number of goods in process is drastically reduced E) Quality of finished products increases Answer: D 143) The production method that brings together all materials and parts needed at each production stage at the precise moment at which they are required is A) master production scheduling. B) just-in-time. C) inventory control. D) warehousing. E) material requirements planning. Answer: B 144) Joe is working with a system that reduces the number of goods in process and keeps the production process flowing smoothly with a bare minimum of inventory. The system is most likely A) material requirements planning. B) warehousing. C) inventory control. D) just-in-time. E) master production scheduling. Answer: D 145) The production control method in which a bill of materials is used to ensure that the right amounts of materials are delivered to the right place at the right time is A) material requirements planning. B) master production scheduling. C) just-in-time. D) inventory control. E) warehousing. Answer: A 146) A very popular process among companies whose products require complicated assembly and fabrication activities is A) inventory control. B) material requirements planning. C) just-in-time. D) master production scheduling. E) warehousing. Answer: B 147) Bill is working with a system that uses a bill of materials to project the number of components and raw materials needed. It also specifies when they need to arrive. He is most likely using A) warehousing. B) master production scheduling. C) material requirements planning. D) just-in-time. E) inventory control. Answer: C 148) All of the following are main areas of materials management except A) transportation. B) operations control. C) warehousing. D) inventory control. E) purchasing. Answer: B 149) The management of the production process designed to manufacture goods or supply services that meet specific quality requirements is called A) just-in-time. B) inventory control. C) quality control. D) material requirements planning. E) master production scheduling. Answer: C 150) ________ is a measure of economic performance that compares how much is produced relative with the resources used to produce it. A) Re-engineering B) Services C) Total quality management D) Productivity E) Quality Answer: D 151) Productivity measures how much we produce relative to the ________ we use to produce it. A) labour B) time C) resources D) materials E) dollars Answer: C 152) Which of the following is the best description of quality? A) A product's fitness for use by consumers B) How closely a product adheres to quality standards C) A measurement of how much is produced relative to the resources used to produce the product or service D) The additional output in the current year compared to the output for the previous year E) A measurement of how much is produced Answer: A 153) If the total resources needed to create a product decrease, productivity has A) decreased and quality has increased. B) not changed. C) decreased. D) cannot determine from information provided. E) increased. Answer: E 154) If a nation's productivity drops as compared to competing nations, it will experience A) low inflation. B) a fall in its standard of living. C) higher profits. D) no loss of income. E) increased wealth. Answer: B 155) The 2008 OECD study showed that productivity was highest in _________ and lowest in _________. A) Belgium; Australia B) The United States; China C) China; The United States D) Luxembourg; Chile E) Japan; France Answer: D 156) In the 2008 OECD study, Canada ranked ______ among the 30 countries studied. A) 1st B) 3rd C) 6th D) 17th E) 30th Answer: D 157) Economic output per hour worked differs from nation to nation. This is due to all of the following factors except A) economic policies. B) size of the nation. C) human skills. D) traditions. E) natural resources. Answer: B 158) Which of the following is not a result of declining productivity? A) Foreign competition will increase B) Firms can afford to increase the wages of workers C) A nation's total wealth will shrink D) A declining standard of living E) One person's increase in wealth will come only at the expense of another person in the society Answer: B 159) According to Michael Porter, a Harvard University expert on international competitiveness, Canada's competitiveness is a concern because A) Canada's productivity is the lowest in the world. B) we have been living off our rich diet of natural resources. C) Canadian companies are only competitive due to the lower value of the Canadian dollar. D) the quality of Canadian manufactured goods is low. E) we don't take advantage of the intellectual ability of our people. Answer: B 160) Which of the following is not a feature of a business firm with high productivity? A) The standard of living of its employees will be lowered B) Greater profits may be made on each item sold C) Greater competitive advantage is created D) Products may be offered at a lower price E) Costs are lower Answer: A 161) Which of the following would have the least interest in the productivity level of a Canadian business firm? A) A foreign firm not in direct competition with the firm B) Employees of the firm C) Investors D) Union officials and members E) Managers Answer: A 162) Which of the following is an advantage to a firm with increasing productivity? A) Machine breakdowns will be less likely B) Raw materials will decline in price C) The firm can afford to pay workers higher wages without raising prices D) Workers will not want to unionize E) The standard of living of its employees will fall Answer: C 163) Productivity is measured primarily by using A) government estimates. B) comparisons with competitors. C) balance sheet statements. D) ratios. E) profit or loss amounts. Answer: D 164) An artist over the period of a day converts 8 lbs. of steel bought at $10/lb. and 2 lbs. of plastic bought at $5/lb. into 4 lamps that he sells for $50 per lamp. What is the productivity for that artist over the period of one day? A) 90 B) 2.22 C) 200 D) 120 E) 4 Answer: B 165) An artist over the period of a day converts 8 lbs. of steel bought at $10/lb. and 2 lbs. of plastic bought at $5/lb. into 4 lamps that he sells for $50 per lamp. What is the artist's material productivity per pound of steel used over the period of one day? A) $25 B) 1/2 lamp C) $40 D) $90 E) 2 lamps Answer: B 166) Suppose that you are required by your boss to identify how many units of product are produced relative to the resources used. What following measure will allow you to do this? A) Quotas B) Competition C) Growth rate D) Productivity E) Quality Answer: D 167) Which term identifies the amount produced relative to the resources used to produce it? A) Quotas B) Tariffs C) Quality D) Reverse output ratio E) Productivity Answer: E 168) Which of the following reflects the value of the resources used to create the outputs? A) Inputs B) Quality C) Outputs D) Transformation E) Productivity Answer: A 169) Labour productivity of a country is a ratio calculated by dividing ________ by _________. A) productivity; quality B) GNP; the total number of workers C) GDP; the total number of workers D) inputs; outputs E) outputs; inputs Answer: C 170) In order to compute labour productivity, data must be available on what two factors? A) GDP and unemployment rates B) GNP and total output C) GDP and the total number of workers D) Unemployment rates and inflation rates E) None of these are correct Answer: C 171) If a country’s labour force is 200 million people, and its GDP is $500 million, what is its labour productivity? A) $2.50 B) $0.40 C) $700 D) $300 E) it is not possible to compute the answer from the information given Answer: A 172) Which of the following statements about productivity is correct? A) Productivity does not vary much between countries B) It is difficult to compute productivity for different countries because of different industrial practices C) Additional wealth from higher productivity cannot be shared among workers D) Manufacturing productivity is higher than service productivity E) It is difficult to increase productivity in the service sector Answer: D 173) When GDP is divided by the total number of workers in a country, ________ can be determined. A) materials productivity B) total productivity C) total factor productivity D) industry productivity E) labour productivity Answer: E 174) When a firm buys a new fleet of trucks, or a new state-of-the-art machine, this is likely to increase ________ productivity. A) output B) capital C) labour D) materials E) motivational Answer: C 175) Both productivity and quality improvements consider A) knowledge of current cost levels. B) paying attention to customer needs. C) total productivity ratios and quality/cost studies. D) competitive product analysis. E) statistical process control. Answer: B 176) In Michael Porter's view, Canada will have to start emphasizing ________ if it hopes to be successful in international markets. A) hard work B) innovation C) growth D) recycling E) total quality management Answer: B 177) Levels of productivity refer to A) the “fitness” of a product for use by customers. B) the dollar value of goods and services produced by each worker in a country. C) profit or loss amounts by all industrial companies in a given country. D) government estimates of the value of goods and services produced in a country. E) comparisons with competitor countries. Answer: B 178) If England's productivity drops as compared to competing nations, it will experience A) no loss of income. B) higher profits. C) a decrease in its standard of living. D) increased wealth. E) low inflation. Answer: C 179) Which of the following individuals is credited with helping Japanese manufacturers achieve a dramatic improvement in quality? A) Shoichiro Toyoda B) Abraham Maslow C) Malcolm Baldridge D) W. Edwards Deming E) Frederick W. Taylor Answer: D 180) Who developed "cause-and-effect diagrams"? A) Kaoru Ishikawa B) Malcolm Baldridge C) Shoichiro Toyoda D) W. Edwards Deming E) Joseph Juran Answer: A 181) ___________, which is used to help track down what is causing quality problems, is also known as __________. A) The Ishikawa diagram; the fishbone diagram B) Total quality assurance; quality control C) Company productivity; department productivity D) Value-added analysis; statistical process control E) Quality improvement; the quality circle Answer: A 182) "Fishbone diagrams" are designed to A) show the steps in the production process. B) measure the amount of process variation. C) identify management steps for ensuring quality. D) help teams of employees investigate and track down causes of quality problems in their work areas. E) identify the inefficiency problems in the production process. Answer: D 183) The starting point for total quality management is A) quality ownership. B) performance quality. C) quality reliability. D) customer focus. E) value-added analysis. Answer: D 184) The features that are designed into a product are a function of A) quality circles. B) quality control. C) performance quality. D) total quality management. E) quality reliability. Answer: C 185) The concept that emphasizes that no defects are tolerable and that all employees are responsible for maintaining quality standards is called A) performance quality. B) total quality management. C) quality reliability. D) quality ownership. E) quality circle. Answer: B 186) Total Quality Management considers all parts of the business except A) front line workers. B) senior managers. C) suppliers. D) customers. E) auditors. Answer: E 187) The consistency of product quality from one finished product to the next is A) quality reliability. B) quality control. C) total quality management. D) performance quality. E) quality circles. Answer: A 188) When Maytag gets premium prices for its appliances because they are perceived as offering more advanced features and longer life than competing brands, they are using A) quality control. B) quality reliability. C) quality ownership. D) quality assurance. E) performance quality. Answer: E 189) When Custom Research delivers 99 percent of its final research reports on time it is practising A) performance quality. B) quality reliability. C) quality control. D) quality assurance. E) quality ownership. Answer: B 190) When new versions of software include features not in previous editions, the vendors are practising A) quality assurance. B) quality ownership. C) quality reliability. D) quality control. E) performance quality. Answer: E 191) Which term captures the idea that quality belongs to each person who creates it while performing a job? A) Total quality management B) Value added analysis C) Employee empowerment D) Competitive product analysis E) Quality ownership Answer: E 192) Which term refers to all the activities necessary for getting high-quality goods and services into the marketplace? A) Quality circles B) Quality ownership C) Statistical process control D) Total quality management E) Statistical process analysis Answer: D 193) The idea that everyone from the president down to part-time clerks is responsible for quality is the basis for the ________ concept. A) total quality management B) statistical quality control C) quality control D) kaizen E) quality/cost studies Answer: A 194) TQM starts by focusing on which of the following parts of the business? A) Employees B) Customers C) Suppliers D) Competitors E) Investors Answer: B 195) ________ refers to the features of a product and how well it performs. A) Quality reliability B) Quality circles C) Performance quality D) Total quality management E) Quality control Answer: C 196) The consistency of product quality from one finished product to the next is A) productivity. B) re-engineering. C) quality reliability. D) total quality management. E) performance quality. Answer: C 197) The features that are designed into a product are a function of A) performance quality. B) total quality management. C) quality control. D) quality reliability. E) quality circles. Answer: A 198) The idea that quality belongs to each person who creates it while performing a job is known as A) total quality management. B) value added analysis. C) quality ownership. D) competitive product analysis. E) employee empowerment. Answer: C 199) Which of the following is an effective method for improving success in quality efforts? A) Train quality inspectors in inspection techniques B) Hire better trained and qualified employees C) Establish a separate quality control department D) Distribute quality inspectors who will inspect every component of the product E) Use statistical process control methods throughout the production process Answer: E 200) Which is the best description of competitive product analysis? A) A company looks at all the competitors' products and rates them on a scale of 1 to 10 B) An independent testing service, such as Consumers' Union, is hired to test and rate a group of similar products C) A competitor's product is purchased, disassembled, and the components tested in order to generate ideas for improving the quality of products D) An independent firm conducts consumer research in order to rate competing products E) This is the result of a consumer survey by the marketing department Answer: C 201) While some process variation may be acceptable, too much process variation can result in A) lower equipment costs. B) higher profits. C) customer satisfaction. D) poor quality. E) cost savings. Answer: D 202) Which of the following is not an example of an external failure? A) Refunds to customers B) Transportations costs to return defective products C) Factory recalls D) Raw materials costs E) Possible lawsuits Answer: D 203) If Toshiba buys a Xerox copier and then takes it apart to determine how well Toshiba's components perform compared to Xerox's, this is an example of A) statistical process control. B) competitive market research. C) process capability. D) competitive product analysis. E) quality ownership. Answer: D 204) The process by which a company analyzes a competitor's product to identify desirable improvements in its own is called A) quality reliability analysis. B) performance quality analysis. C) quality/cost studies. D) competitive product analysis. E) benchmarking. Answer: D 205) The evaluation of all work activities, materials flows, and paperwork to determine the value that they add for customers is A) value-added analysis. B) quality/cost studies. C) statistical process control. D) getting closer to the customer. E) benchmarking. Answer: A 206) What is the purpose of value-added analysis? A) To determine how much each component part in a product costs B) To determine how productive assembly line workers are C) To determine how high the quality of a product is D) To determine the value that work activities add for customers E) None of these Answer: D 207) When Hewlett-Packard simplified its contracts and reduced them from 20 pages to two pages, it was engaging in A) quality/cost studies. B) statistical process control. C) getting closer to the customer. D) value-added analysis. E) benchmarking. Answer: D 208) The evaluation method that allows managers to analyze variations in a company's production activities to determine when adjustments are needed is called A) internal failure analysis. B) statistical process control. C) quality circles. D) upper/lower control limit analysis. E) quality/cost study. Answer: B 209) When analysts at Farbex spot-check the weight of samples at several points in the production process they are engaging in A) value-added analysis. B) quality/cost studies. C) benchmarking. D) statistical process control. E) getting closer to the customer. Answer: D 210) What is the purpose of statistical process control? A) To analyze variations in department productivity B) To analyze variations in department quality C) To analyze variations in production data D) To identify a firm’s current costs E) To compare the firm against its key competitors Answer: C 211) Which of the following is not a process variation? A) Work methods B) Equipment C) Employees D) Customers E) Materials Answer: D 212) Which of the following is not a part of statistical process control methods? A) Control limits B) Process variation C) Quality circles D) Control charts E) Specification limits Answer: C 213) A chart on which results of test sampling of a product are plotted to reveal when a process is beginning to depart from normal operating conditions is known as a(n) A) Gantt chart. B) productivity chart. C) control chart. D) kaizen chart. E) upper specification limit chart. Answer: C 214) George is developing information about the amount of variation in the production of electronic components. He is doing A) a value-added analysis. B) benchmarking. C) a quality study. D) a process capability study. E) a specification study. Answer: D 215) Which of the following provides the quality limits for acceptable products? A) Quality cost studies B) Competitive product analysis C) Internal failure analysis D) Upper and lower specifications limits E) Control chart Answer: D 216) Which of the following provides information about variation in a process? A) Measurement calibration B) Specification limits C) The machine operator D) A process capability study E) A quality circle Answer: D 217) What is the purpose of a process capability study? A) To determine how the quality of the company’s products compares to the quality of its competitors’ products B) To provide information about variation in the company’s production processes C) To determine how much value various work activities add value for customers D) To determine if work teams will improve quality E) To determine whether the causes of product failures are internal or external Answer: B 218) Louis is a production manager. How can he determine if production machinery is capable of meeting the upper and lower specification limits? A) Use control charts B) Conduct a process capability study C) Conduct competitive product analysis D) Implement a quality circles program E) Hire a marketing consultant Answer: B 219) Which of the following will determine if production machinery is capable of meeting the upper and lower specification limits? A) Quality circles B) Competitive product analysis C) Value-added analysis D) Control charts E) A process capability study Answer: E 220) Which of the following is correct with regard to setting the control limits for the control chart? A) Control limits depend solely on statistical considerations B) Quality circles determine the control chart's control limits C) The control chart is used to plot the result of individual units which were sampled D) The control limits are set as 5% higher than the specification limits E) The control limits are the same as the specification limits for individual units Answer: A 221) Pauline is an operator monitoring a production process. Which one of the following should she use during the production process to monitor quality of the product? A) A process capability study B) A control chart C) Competitive product analysis D) A pacing meter system E) Quality circles Answer: B 222) If managers want to increase the uniformity of their outputs by understanding the sources of variation, they would use A) statistical process control. B) internal failure analysis. C) quality/cost study. D) quality circles. E) upper/lower control limit analysis. Answer: A 223) Harry needs to receive accurate information about variation in the production process, so he should do a A) machine operator study. B) measurement calibration. C) process capability study. D) benchmarking study. E) control chart. Answer: C 224) A consultant who came to study the production process at Honey Nuggets Inc. determined the accuracy of the upper and lower specification limits by A) a process capability study. B) a competitive product analysis. C) instituting a quality circles program. D) a triangulation process. E) developing a control chart. Answer: A 225) Which of the following is an internal failure that should be considered in a quality/cost study of Honey Nuggets cereal? A) How much the firm spends for appraising quality during production B) Expenses incurred during production before bad products leave the plant C) How much the firm spends to prevent production of defective products D) Expenses incurred after bad products reach the consumer E) All of these Answer: B 226) The filling machine operator at Honey Nuggets weighs a box of cereal from the production line every 15 minutes. Where is she likely to record the data? A) Control charts B) Benchmarking C) Process capability studies D) Value-added analysis E) Quality/cost studies Answer: A 227) At the Honey Nuggets factory, boxes are taken from each of three filling machines. Analyzing the results show that one of the machines cannot reliably meet specification. Which tool are they using? A) Value-added analysis B) Control charts C) Process capability studies D) Quality/cost studies E) Benchmarking Answer: C 228) Ken is a production operator who plots a sample of the product and finds that it is outside the upper or lower specification limits on the control chart. What should he do? A) Continue with production and analyze why the problem might be occurring B) Do nothing; wait to see if the next sample is within the control limits C) Tell co-workers “the sky is falling”! D) Correct the problem (e.g., people, equipment, materials, or work methods) E) Shut down the machine and have a manager investigate the problem Answer: E 229) Quality/cost studies show what a firm's current costs are, and also reveal A) areas that have the biggest cost-savings potential B) the limits of acceptable and unacceptable quality in production of a good or service. C) how much quality will have to be achieved in order to meet consumer expectations. D) how motivated employees are to build quality into products. E) the level of quality that is currently being achieved. Answer: A 230) Which term identifies the results of a quality/cost study that identified defective products that reached customers? A) External failures B) Boundary-spanning failures C) Six sigma D) Internal failures E) Quality circles Answer: A 231) Which term identifies the results of a quality/cost study that identified expenses incurred during production before unacceptable products leave the plant? A) External failures B) Six sigma C) Quality circles D) Boundary-spanning failures E) Internal failures Answer: E 232) When a company analyzes and applies the best practices of other companies with whom they do not directly compete to their own operations, this is A) quality/cost analysis. B) performance quality analysis. C) benchmarking. D) quality reliability analysis. E) competitive product analysis. Answer: C 233) With ________, a firm tracks its own performance over time to evaluate progress and set goals for improvement. A) quality improvement B) failure management C) process improvement D) external benchmarking E) internal benchmarking Answer: E 234) All of the following are costs of correcting external failures except A) possible lawsuits. B) refunds to customers. C) factory recalls. D) higher wages. E) transportation costs to return bad boxes to the factory. Answer: D 235) Which of the following is an external failure that should be considered in a quality/cost study of Honey Nuggets cereal? A) The cost of overfilling boxes B) Expenses incurred during production before bad products leave the plant C) The cost of sorting out bad boxes D) Expenses incurred after bad products reach the consumer E) All of these Answer: D 236) __________ occurs when a company compares its current performance against its own past performance, while _________ occurs when a company compares its current performance against the performance of its competitors. A) Internal productivity; external productivity B) Internal benchmarking; external benchmarking C) Internal benchmarking; getting close to the customer D) Statistical process control; external productivity E) Internal quality/cost study; external quality/cost study Answer: B 237) ________ means analyzing a competitor's product to identify desirable improvements, while __________ means evaluating paperwork to determine the value it adds for customers. A) Value added analysis; benchmarking B) Benchmarking; re-engineering C) Competitive product analysis; value added analysis D) Statistical process control; performance quality analysis E) Value added analysis; quality reliability analysis Answer: C 238) When Motorola wanted to reduce the time between receipt and delivery of a customer's order for cellular phones, it studied Domino's Pizza procedures for delivery cycles. This is an example of A) quality/cost studies. B) benchmarking. C) quality reliability analysis. D) performance quality analysis. E) competitive product analysis. Answer: B 239) Which of the following is not a function of quality improvement teams? A) Solve the problem B) Gather data to evaluate the severity of problems C) Analyze alternative solutions for the problem D) Define problem areas E) All of the above are functions of quality improvement teams Answer: E 240) The process of comparing the quality of the firm's output with the quality of the output of industry leaders is called A) quality/cost studies. B) competitive product analysis. C) performance quality analysis. D) quality reliability analysis. E) benchmarking. Answer: E 241) Through ________, a firm compares its products and processes with those of competitors or even companies in other lines of business. A) internal benchmarking B) external benchmarking C) process improvement D) quality improvement E) failure management Answer: B 242) According to one expert, " ________ are an economic asset. They're not on the balance sheet, but they should be." A) quality employees B) capital goods C) quality improvement teams D) customers E) competent top management personnel Answer: D 243) The quality program that is being adopted by many multinational firms is A) QC 2000. B) ROQ. C) ISO 9000. D) IQS 8000. E) TQM 2001. Answer: C 244) ISO 9000:2000 certified businesses must A) have documented procedures for each stage of production. B) have no defects. C) have strict quality controls. D) produce a variety of quality goods. E) be productive. Answer: A 245) ISO 9000 standards allow firms to demonstrate that they follow documented procedures for all of the following except A) fixing product defects. B) re-engineering. C) testing products. D) training workers. E) keeping records. Answer: B 246) For ISO 9000 certification firms are required to demonstrate documented procedures for all of the following except A) testing products. B) selecting vendors. C) training workers. D) fixing product defects. E) keeping records. Answer: B 247) ISO 9000 is A) the concept that all employees are valuable contributors to a firm's business, and should be entrusted with decisions regarding their work. B) a program certifying that a factory, laboratory, or office has met the quality management standards of the International Organization for Standardization. C) the principle of looking at the supply chain as a whole in order to improve the overall flow through the system. D) a certification program attesting to the fact that a factory, laboratory, or office has improved environmental performance. E) the redesigning of business processes to improve performance, quality, and productivity. Answer: B 248) In ISO 9000:2000, which of the following is not required for certification? A) Proving that the company has sound financial practices B) Documented procedures for testing products C) Fixing product defects D) Keeping records E) Testing products Answer: A 249) ISO 14000 is A) the concept that all employees are valuable contributors to a firm's business, and should be entrusted with decisions regarding their work. B) the redesigning of business processes to improve performance, quality, and productivity. C) a program certifying that a factory, laboratory, or office has met the quality management standards of the International Organization for Standardization. D) a certification program attesting to the fact that a factory, laboratory, or office has improved environmental performance. E) the principle of looking at the supply chain as a whole in order to improve the overall flow through the system. Answer: D 250) The ________ program certifies improvements in environmental performance. A) IQS 8000 B) TQM 2001 C) ISO 14000 D) QC 2000 E) ISO 9000 Answer: C 251) __________ is a program that certifies improvements in environmental performance, while _________ is a certification program attesting to the fact that a factory has met rigorous quality management requirements A) ISO 9000; ISO 14000 B) Benchmarking; a quality/cost study C) ISO 14000; ISO 9000 D) Environmental statistical process control; ISO 9000 E) Getting close to the customer; a quality control study Answer: C 252) All of the following are components of ISO 14000 except A) supply chain management. B) pollution management. C) identification, treatment, and disposal of hazardous waste. D) environmental labeling of products as "energy efficient" and "recyclable." E) development of an environmental management system. Answer: A 253) ________ focuses on improving both the quality and productivity of an organization's operations. A) ROQ B) Reengineering C) Back to Basics D) Six Sigma E) ISO 9000 Answer: B 254) Business process reengineering is A) the principle of looking at the supply chain as a whole in order to improve the overall flow through the system. B) a program certifying that a factory, laboratory, or office has met the quality management standards of the International Organization for Standardization. C) a certification program attesting to the fact that a factory, laboratory, or office has improved environmental performance. D) the redesigning of business processes to improve performance, quality, and productivity. E) the concept that all employees are valuable contributors to a firm's business, and should be entrusted with decisions regarding their work. Answer: D 255) The process which focuses on both productivity and quality and which entails rethinking by starting over from scratch is A) re-engineering. B) ISO 9000. C) six sigma. D) back to basics. E) ROQ. Answer: A 256) ________ is dominated by a desire to improve operations in order to produce goods and services at the lowest possible cost, and at the highest value for the customer. A) Back to Basics B) ISO 9000 C) Six Sigma D) ROQ E) Re-engineering Answer: E 257) The process which focuses on improving business processes by rethinking each step in a process is called A) business process re-engineering. B) ISO 9000. C) ROQ. D) back to basics. E) six sigma. Answer: A 258) When Caterpillar Financial Services changed the whole payments and financing process by improving equipment, retraining employees, and connecting customers to the company’s data base, this was an example of A) business process re-engineering. B) ISO 9000. C) getting close to the customer. D) back to basics. E) six sigma. Answer: A 259) _________ is focused on improving the overall flow through a system composed of various companies working together. A) Supply chain management B) Flow-through analysis C) Benchmarking D) Business process re-engineering E) Materials management Answer: A 260) When fashion house Louis Vuitton revamped its system in order to ensure that retailers would always have “hot” items available, this was an example of A) supply chain management. B) flow-through analysis. C) benchmarking. D) business process re-engineering. E) marketing research. Answer: A 261) The ________ is the group of companies and stream of activities that work together to create a product. A) value chain B) supply chain C) service channel D) distribution chain E) supply network Answer: B 262) The supply chain typically excludes A) customers. B) retailers. C) suppliers. D) distributors. E) investors. Answer: E 263) Supply chain management is A) the principle of looking at the supply chain as a whole in order to improve the overall flow through the system. B) a program certifying that a factory, laboratory, or office has met the quality management standards of the International Organization for Standardization. C) a certification program attesting to the fact that a factory, laboratory, or office has improved environmental performance. D) the concept that all employees are valuable contributors to a firm's business, and should be entrusted with decisions regarding their work. E) the redesigning of business processes to improve performance, quality, and productivity. Answer: A TRUE/FALSE QUESTIONS. Write 'T' if the statement is true and 'F' if the statement is false. 264) Firms that make tangible products (radios, newspapers, buses, textbooks) are engaged in service operations. Answer: False 265) Although companies are often classified as either goods producers or service providers, the boundary is often blurred. Answer: True 266) The four kinds of utility provided by a product or service are time, place, price, and form. Answer: False 267) Place utility refers to the location where a product is made. Answer: False 268) Operations management is the systematic direction and control of the processes that transform resources into finished goods and services. Answer: True 269) Operations managers do not always work in production facilities; farmers, for example, can be considered operations managers. Answer: True 270) Production managers are relatively easy to identify because almost all of them work in factories. Answer: False 271) The operations transformation process involves the management of inputs that are transformed into outputs. Answer: True 272) "Place utility" refers to the quality of a product satisfying a human want when it is conveniently available for customers. Answer: True 273) When a company turns out ornaments in time for Christmas, this is an illustration of ownership utility. Answer: False 274) All service businesses must locate close to their clients. Answer: False 275) The three characteristics of services that separate them from goods production are intangibility, customization, and storability. Answer: False 276) The three characteristics of services that distinguish them from goods production are intangibility, customization, and unstorability. Answer: True 277) One way to classify a production process is by the type of technology used in the transformation process. Answer: True 278) Fabrication processes involve the alteration of the physical shapes of the raw materials. Answer: True 279) An analytic process combines raw materials to produce a final product. Answer: False 280) A synthetic process combines raw materials to produce a final product. Answer: True 281) Transforming raw data into management reports is an analytic process. Answer: False 282) Service processes are classified according to the extent of the customer contact. Answer: True 283) Utilities and auto repair shops are good examples of low contact processes. Answer: True 284) In low-contact systems customers receive a service without ever coming in direct contact with the system. Answer: True 285) A graphic design company for toilet paper packaging is considered a low-contact process, as there is no direct customer contact. Answer: True 286) A public transit system is an example of a low-contact system. Answer: False 287) Selecting the kind of production that is best for the company is best driven from above by the firm’s business strategy. Answer: True 288) Toyota’s top priority focuses on low-prices, so a cost-minimization strategy is appropriate for the company. Answer: False 289) The long range operations plan should address both capacity and facilities. Answer: True 290) Capacity planning means ensuring that a firm's capacity exactly meets the normal demand for its product. Answer: False 291) In high contact systems capacity should be based on peak demand. Answer: True 292) Alternatives for layout planning include process, product, and cellular. Answer: True 293) Product layouts are efficient ways to manage the manufacturing of multiple product lines because of its inherent structural simplicity. Answer: False 294) The assembly line is a special form of product layout. Answer: True 295) In the product layout of facilities planning, the resources are processed through the same fixed sequence of steps to become finished goods. Answer: True 296) Soft manufacturing emphasizes computer software and computer networks instead of production machines. Answer: True 297) In manufacturing, managers can work to reduce waste and inefficiency by examining procedures on a step-by-step basis, an approach sometimes called method improvement. Answer: True 298) The process flow chart identifies the sequence of production activities, movements of materials, and work performed at each stage as the product flows through production. Answer: True 299) The master production schedule reflects which goods will be produced, when production will occur and what resources will be used in the coming months. Answer: True 300) The adoption of JIT usually means that a company has higher inventory costs than it did before, but a better and larger selection of inventory items. Answer: False 301) Operations control requires production managers to monitor production performance by comparing results with detailed plans and schedules. Answer: True 302) Materials management involves production control, production planning, and organizing the flow of materials. Answer: True 303) The process of materials management not only controls but also plans and organizes the flow of materials. Answer: True 304) Inventory control and materials management have the same meaning and are used interchangeably. Answer: False 305) Material requirements planning uses the product bill of materials for planning production. Answer: True 306) Material requirements planning is the method that brings together all materials and parts needed at each production stage at the precise moment they are needed. Answer: True 307) Productivity is defined as fitness for the consumer's use. Answer: False 308) Higher productivity will generally lead to more and higher quality goods produced. Answer: False 309) Productivity considers both the amounts and the quality of what is produced. Answer: True 310) Quality means a product's fitness for use in terms of offering the features that consumers want. Answer: True 311) Productivity is measured by a ratio of inputs to quality. Answer: False 312) Labour productivity is equal to the gross national product divided by the total number of workers. Answer: False 313) A country's labour productivity is usually calculated as gross domestic product divided by the total number of workers. Answer: True 314) A decline in productivity shrinks a nation's available resources so that any one person's increase in wealth can come only at the expense of others in the society. Answer: True 315) Firms with increasing productivity rates can provide higher wage increases more easily than firms with declining productivity rates. Answer: True 316) Quality assurance includes all the activities necessary for getting high-quality goods and services into the marketplace. Answer: True 317) Performance quality is the consistency of product quality from unit to unit. Answer: False 318) Planning for quality begins before products are initially designed or redesigned. Answer: True 319) In planning for quality, managers must set goals for both performance quality and quality reliability. Answer: True 320) Applying the concept of quality ownership results in an increase in the power and authority of the quality control department and their inspectors. Answer: False 321) Quality ownership is the concept that quality belongs to each person who creates it while performing a job. Answer: True 322) The two most common statistical process control methods are process-variation studies and control charts. Answer: True 323) Statistical process control methods allow managers to analyze variations in production. Answer: True 324) A process capability study provides information about variation in a production process. Answer: True 325) Information about variation in a production process is obtained from a process capability study. Answer: True 326) Control charts are used to monitor the process during production in order to detect the time at which the process becomes unacceptable. Answer: True 327) Quality/cost studies are the evaluation of all work activities, materials flows, and paperwork to determine the value that they add for customers. Answer: False 328) External failures in quality/cost studies focus on costs incurred during production. Answer: False 329) Internal failures in quality/cost studies focus on costs incurred during production and before bad products leave the plant. Answer: True 330) Benchmarking is the process of finding, analyzing, and applying the best practices of other companies, with whom they may or may not compete, to their own operations. Answer: True 331) Quality improvement teams are composed of employees who work in teams to define, analyze, and solve common production problems. Answer: True 332) I.S.O. stands for "International Standardization of Operations." Answer: False 333) Re-engineering is the process by which a company implements the best practices from its own past performance and those of other companies to improve its own products. Answer: False 334) The goal of supply chain management is to reduce the number of links in the channel. Answer: False 335) Supply chain management focuses on the earliest parts of the supply chain in order to discover economies of operation. Answer: False SHORT ANSWER QUESTIONS. Write the word or phrase that best completes each statement or answers the question. 336) What is the difference between production and operations? Answer: The term production historically meant the making of physical goods. But consumers also purchase a wide variety of services. The operations is now typically used to refer to the making of physical goods and the provision of services. 337) What is utility? Answer: Utility is a product's ability to satisfy a human want. 338) Explain time, place, and form utility. Answer: Time utility is created when a company makes products available when consumers want them. Examples may include seasonal items such as Christmas ornaments, bathing suits, and mosquito repellant. Place utility is created when a company makes product availability convenient for consumers. An example is the placement of routinely purchased items in convenience stores or conveniently located discount stores. Form utility is created when a company combines materials to create products. 339) Identify the three qualities that distinguish services from goods. Answer: Service qualities include intangibility (the service cannot be touched, tasted, or smelled) , customization (the service is matched to the customer’s specific needs) , and unstorability (the service cannot be produced ahead of time and then stored). 340) Explain why services are typically characterized by a high degree of unstorability. Answer: Services usually can't be produced ahead of time and then stored for high-demand periods. If a service isn't used when available, it's usually wasted. 341) Give an example of a product that is produced through an analytic process. Answer: (Answers will vary.) The text gives the example of whole chickens being processed into packaged parts. An example of a product produced through an analytic process is pharmaceutical drugs, which require extensive chemical analysis and testing for efficacy and safety. 342) Give an example of a product that is produced through a synthetic process. Answer: (Answers will vary.) The text uses the example of the production of refrigerators. One example of a product produced through a synthetic process is nylon. Nylon is a synthetic polymer that was first produced by Wallace Carothers at DuPont in 1935. It is made through a condensation reaction between diamines and dicarboxylic acids or by ring-opening polymerization of lactams. Nylon has various applications due to its durability, elasticity, and resistance to abrasion and chemicals. It is commonly used in textiles (such as clothing and carpets), as well as in industrial applications like ropes, fishing nets, and automotive parts. The synthetic production of nylon exemplifies how chemical processes can create materials with specific properties tailored to meet diverse technological and industrial needs. 343) What is the difference between high-contact and low-contact services? Give an example of each. Answer: A high-contact service is one in which the service cannot be provided unless the customer is physically in the system (for example, hair salons, public transportation). A low-contact service is one in which the service can be provided without the customer being physically in the system (for example, lawn care, cheque processing). 344) How does a company select a production strategy? Answer: Companies select the kind of production that best fits with its larger business strategy. 345) The text describes Toyota's top priority as focusing on quality. Explain how Toyota's operations capability would match up with its business strategy if its top priority had been to compete as the low-price car in the industry. Answer: If Toyota's business strategy had been to be the low-cost car maker in the industry, Toyota's operations would obviously focus on cost minimization, not quality. The drive to lower costs would have influenced many strategic decisions, such as inventory policies, product assortment, location of manufacturing facilities, labour costs. The same general types of strategic decisions that were made by Save-A-Lot (which is pursuing a low-cost strategy) would have been adopted by Toyota. 346) What is a forecast? Answer: A forecast is a prediction about future demand. 347) What is the first step in the operations planning and control process? Answer: Business plan and forecasts. 348) What is the last step in the operations planning and control process? Answer: Output to customers. 349) Identify the primary factor influencing the location of high-contact services. Answer: High-contact services must be located close to customers. 350) List the three types of space in a goods-producing facility. Answer: Productive facilities, non-productive facilities, and support facilities. 351) What is a process layout? Answer: A process layout is a spatial arrangement of production activities that groups equipment and people according to function. 352) What is a cellular layout? Answer: A cellular layout is a spatial arrangement of production facilities designed to move families of products through similar flow paths. 353) What is a product layout? Answer: A product layout is a spatial arrangement of production activities designed to make one type of product in a fixed sequence. 354) What is a flexible manufacturing system? Answer: A flexible manufacturing system is a production system in which a single factory uses automation to produce a wide variety of products. 355) What is a master production schedule? Answer: A master production schedule is a production system in which a single factory uses automation to produce a wide variety of products. 356) Identify the five major areas of materials management. Answer: Transportation, warehousing, inventory control, supplier selection, and purchasing. 357) What are the four major tools in production process control? Answer: Worker training, lean production systems, material requirements planning, and quality control. 358) What is productivity? Answer: Productivity is the measure of economic performance that compares how much a system produces with resources needed to produce it. 359) What is quality? Answer: Quality is a product's fitness for use and success in offering features that consumers want. 360) What is the formula for labour productivity for a country? Answer: Dividing gross domestic product by the total number of workers yields labour productivity. 361) What is total quality management? Answer: Total quality management is the sum of all activities involved in getting high-quality products into the marketplace. 362) What is value-added analysis? Answer: Value-added analysis is the process of evaluating all work activities, material flows, and paperwork to determine the value they add for customers. 363) What is statistical process control? Answer: Statistical process control is a method for gathering data to analyze variations in production activities to see when adjustments are needed. 364) What are two commonly used statistical process control tools? Answer: Process variation studies and control charts. 365) What is a control chart? Answer: A control chart is a process control method that plots test-sampling results on a diagram to determine when a process is beginning to depart from normal operating conditions. 366) What is a quality/cost study? Answer: A quality/cost study is a method of improving quality by identifying current costs and areas with the greatest cost-saving potential. 367) Give an example of an internal failure. Answer: (Answers will vary.) An internal failure is a reducible cost incurred during production and before bad products leave a plant. An example from the text was over- and under-filled boxes of cereal that were discovered before they left the plant. An example of an internal failure is a manufacturing defect discovered during quality control inspections before the product is shipped to customers, leading to rework or scrap. 368) Give an example of an external failure. Answer: (Answers will vary.) An external failure is a reducible cost incurred after defective products have left a plant. These costs may include the costs of correcting failures (refunds to customers, transportation costs to return defective products to the factory, possible lawsuits, and factory recalls). An example of an external failure is a customer complaint about a faulty product received after purchase, necessitating a product recall or warranty replacement. 369) What is a quality improvement team? Answer: A quality improvement team (modeled on Japanese quality circles) is a TQM tool in which groups of employees work together to improve quality. 370) What is benchmarking? Answer: Benchmarking is the process by which a company implements the best practices from its own past performance and those of other companies to improve its own products. 371) What is ISO 14000? Answer: ISO 14000 is a certification program attesting to the fact that a factory, laboratory, or office has improved environmental performance. 372) What is re-engineering? Answer: Re-engineering is the fundamental rethinking and radical redesign of business processes to achieve dramatic improvements in measures of performance, such as cost, quality, service, and speed. 373) What is a supply chain? Answer: The supply chain is the group of companies and stream of activities that work together to create a product. ESSAY QUESTIONS. Write your answer in the space provided or on a separate sheet of paper. 374) Discuss the current concept of "production" including recent trends. Answer: • production now means services as well as goods • increased growth in services • products are a combination of goods and services • factories are clean and high-tech • production requires more flexibility 375) What does it mean to say, "production managers are ultimately responsible for creating utility"? Explain this phrase and the term "utility" in your response. Answer: • the products create utility—want satisfaction; production managers help to create that utility • for instance—time utility, and the production of Christmas lights, ownership utility, place utility, and form utility 376) Explain what "utility" means and identify the four types of utility. Answer: • a product's ability to satisfy a want • types of utility: • time, place, ownership, form 377) Identify and briefly discuss the three characteristics which uniquely separate services from goods. Answer: • intangibility—services cannot be touched, tasted, smelled or seen • customization—services are tailored to meet each individual's needs • unstorability—if a service is not used when it is available, it is usually wasted 378) Describe the ways a business's operation processes can be classified. Answer: • can be classified by the technology (chemical, fabrication, assembly, transport or clerical) • analytic or synthetic processes • pattern of product flow (continuous or intermittent) • extent of labour use 379) Identify and explain the techniques of classifying production processes by the type of transformation technology used. Answer: • chemical: chemical alteration of raw materials • fabrication: mechanical alternation of basic form of product • assembly: putting together various components • transport: where goods acquire place utility moving from one place to the next • clerical processes: transformation of information 380) What is the difference between an analytic process and a synthetic process? Answer: • analytic process—any production process in which resources are broken down • synthetic process—any production process in which resources are combined 381) Services are classified according to the extent of customer contact. Identify and briefly discuss the two segments. Answer: • high-contact system—a system in which the service cannot be provided without the customer being physically in the system (e.g., transit systems, hair styling, restaurants) • low-contact system—a system in which the service can be provided without the customer being physically in the system (e.g., lawn care services, auto repair) 382) What is the relationship between business strategy and operations? Answer: The strategy that the business is pursuing influences how operations will be carried out. For example, Toyota’s strategy is to provide customers with high quality automobiles, and this means that operations activities must emphasize high quality standards and continuous improvement. For additional examples, see Table 11.3. 383) What are the steps involved in operations planning and control? Answer: • business plan and forecasts • long-range operations plan (capacity, location, layout, methods) • operations schedules (master production schedule, detailed schedules) • operations control (quality control, materials management) • output to customers • information results (feedback) 384) Describe the five elements of operations planning. Answer: The five categories of operations planning include: capacity, location, layout, quality and methods planning. Capacity planning depends on how many people are employed and the number and size of the facilities and considers both current and future capacity requirements. If capacity is too small for demand, the company must turn customers away; if capacity greatly exceeds demand, the firm is wasting money by maintaining facilities that are too large. Location planning is crucial because location affects production costs and flexibility. Location often depends on the types of goods and services being produced, proximity to raw materials and supplies, and proximity to a skilled workforce. Layout considers the placement of machinery, equipment, and supplies within the facility. Quality planning involves all operations to ensure that products and services meet the firm's quality standards; quality planning prepares employees for continuously improving the firm's products and production processes. Finally, methods planning allows managers to identify each production step and the specific methods for performing it; waste and inefficiency can be reduced by managers examining production procedures on a step-by-step basis. 385) Layout must be planned for different types of space. Identify and briefly discuss the three different types of space. Answer: • productive facilities—workstations and equipment for transforming raw materials • nonproductive facilities—storage and maintenance areas • support facilities—offices, restrooms, parking lots, cafeterias, etc. 386) Identify and briefly discuss the three types of layout planning. Answer: • process: grouped according to function, each area specialized based on task • product: grouped by sequential steps to produce product • cellular: cells created to handle similar but slightly different subcomponents 387) Explain how custom-products layout is grouped according to function. For example, in the local print shop, they are doing a custom job for a college's annual report. There are pictures to be included, financial information, and an envelope for annual giving, all the while needing to be collated, stapled and mailed. Answer: The main advantage of custom-products layouts is flexibility; at any time, the shop can process individual customer orders, each requiring different kinds of work. They can flow through as many activity areas as needed for their specific custom job. 388) How should capacity planning be done for service businesses? Answer: based on degree of customer contact; high-contact should be based on peak demand; low-contact should be based on average demand 389) Identify and describe the five major areas in materials management. Answer: • transportation includes the means of transporting resources to the company and finished goods to buyers • warehousing is the storage of both incoming materials for production and finished goods for physical distribution to customers • purchasing is the acquisition of all the raw materials and services that a company needs to produce its products • supplier selection means finding and choosing suppliers of services and materials to buy from • inventory control includes the receiving, storing, handling, and counting of all raw materials to buy from 390) Describe how just-in-time (JIT) production systems help managers minimize manufacturing inventory costs. Answer: • just-in-time (JIT) production systems — a method of inventory control in which materials are acquired and put into production just as they are needed • all resources are continuously flowing, from their arrival as raw materials to subassembly, final completion, and shipment of finished products • JIT reduces to practically nothing the number of goods in process and saves money by replacing stop-and-go production with smooth movement • once smooth movements become the norm, disruptions become more visible and thus are resolved more quickly • finding and eliminating disruptions by continuous improvement of production is a major objective of JIT 391) Explain how material requirements planning works in the production department. Answer: • MRP seeks to deliver the right amounts of materials to the right place at the right time • MRP uses a bill of materials as a "recipe" for the finished products • results in: • fewer early arrivals • less frequent stock shortages • lower storage costs • MRP is popular among companies whose products require complicated assembly and fabrication activities • MRP II is an advanced version of MRP, and it ties all parts of the organization into the company's production activities 392) Explain the connection between productivity and quality. What do these two terms mean? Answer: • productivity: a measure of efficiency that compares how much is produced with the resources used to produce it • quality: a product's fitness for use in terms of offering the features that consumers want • quality and productivity are connected because they can affect ability to compete • countries that focus on both efficiency and quality have increased productivity 393) Why is it important that Canada increase its productivity? Answer: • during the 1980s, the growth rate of Canadian productivity slowed down • in recent years, Canada's growth rate of productivity has lagged behind that of countries like Japan, South Korea and France • it is important that Canadian business firms increase their rate of productivity growth so that they can be competitive in world markets as the productivity of Canadian business firms increases, they will be able to produce a greater quantity of goods without using more resources • if Canada loses ground to other countries it will be more difficult to export our products and our standard of living will decline 394) Who is Michael Porter and what is his view on Canada's competitiveness? Answer: • Michael Porter is a Harvard University expert on international competitiveness • He states that Canada's competitiveness is a concern because we have been living off our rich diet of natural resources • Canada will have to start emphasizing innovation and develop a more sophisticated mix of products if it hopes to be successful in international markets • Porter criticizes Canadian business, government, and labour for failing to abandon outdated ways of thinking regarding productivity and innovation 395) How does an economist measure labour productivity? Is there a formula for this measurement? Who provides the data in measuring national productivity? Answer: • an economist measures labour productivity by dividing the gross domestic product by the total number of workers provided by Statistics Canada • labour productivity = gross domestic product/total number of workers • sometimes the total hours worked is substituted for total number of workers 396) How does high productivity give a company a competitive edge? Answer: • High productivity gives a company a competitive edge because it's costs are lower. As a result, it can offer its product at a lower price and gain more customers or it can make a greater profit on each item sold. Increased productivity also allows companies to pay workers higher wages without raising prices. 397) What is TQM and why is it important? Answer: • TQM stands for Total Quality Management • it is a concept that emphasizes that no defects are tolerable and that all employees are responsible for maintaining quality standards 398) How does a business firm plan for quality? Answer: • by setting goals for quality even before the products are designed or redesigned • two aspects: • performance quality: overall degree of quality; how well the features of a product meet customers' needs and how well the product performs • quality reliability: the consistency of quality from unit to unit of a product 399) Describe the tools for quality management and outline how they can be used. Answer: • often, ideas for improving both the product and the production process come from competitive product analysis: the analysis of products made by a competitor ↕ there are eight methods that can be used to ensure quality: • value-added analysis: evaluation of work activities, material flows, and paperwork to determine the value they add for customers • statistical process control: techniques that analyze variations in production data • quality/cost studies: method to determine a firm's current costs and to determine areas with the greatest cost-savings potential • quality circles: technique that groups production employees into teams that deal with quality problems • benchmarking: comparing the quality of the firm's output with those of industry leaders • cause-and-effect diagram: summarizes four possible causes of quality problems—materials, manpower, methods, and machines • ISO 9000: quality scorecard developed by the International Standards Organization • re-engineering: process of rethinking and redesigning business processes 400) How does the use of statistical process control methods serve to improve the quality of a firm's products? Answer: • SPC methods allow managers to analyze variations in production data, i.e. better identify what areas need improvement • two aspects: • process variation: determination that the process has the capability to meet quality standards • control charts: determination that the process is actually meeting quality standards 401) Your boss wants to determine whether or not to apply for ISO 9000. What are the benefits? Answer: • ISO 9000 is a quality scorecard developed by the International Standards Organization • aim is to find the cause of product defects at the production line level • company must document procedures for: • testing products • training workers • keeping records • fixing product defects • companies must be audited • process takes time and money • however, it is quickly becoming a requirement for much international trade and a considerable amount of domestic trade Test Bank for Business Essentials Ronald J. Ebert, Ricky W. Griffin, Frederick A. Starke, George Dracopoulos 9780132479769, 9780134384733

Document Details

Related Documents

Close

Send listing report

highlight_off

You already reported this listing

The report is private and won't be shared with the owner

rotate_right
Close
rotate_right
Close

Send Message

image
Close

My favorites

image
Close

Application Form

image
Notifications visibility rotate_right Clear all Close close
image
image
arrow_left
arrow_right